LSAT and Law School Admissions Forum

Get expert LSAT preparation and law school admissions advice from PowerScore Test Preparation.

 Administrator
PowerScore Staff
  • PowerScore Staff
  • Posts: 8916
  • Joined: Feb 02, 2011
|
#31342
Complete Question Explanation

While most fill-in-the-blank questions on the LSAT fall into the Must Be True category, the stem here makes clear that this is a strengthen question. We want to finish with new information that will make the conclusion (there will be more diseases caused by drug-resistant bacteria if we stop using penicillin) more likely. This one almost feels like a Resolve the Paradox question - why would there be MORE disease when we switch to a more powerful set of antibiotics that kill a wider variety of bacteria and abandon a weaker one with a narrower range of effectiveness? We need to look for an answer that strengthens the conclusion by resolving that problem - something that shows that the switch could cause this harm.

Answer A: This is the correct answer. If the drug-resistant bacteria do better when other bacteria are removed, and if the new antibiotics are more powerful and kill more different types of bacteria, then they would be clearing the path for the remaining drug-resistant bacteria to flourish. This is a case of "the enemy of my enemy is my friend". If the new stuff gets rid of competition, and drug-resistant bacteria do better without competition, then they will do better when the new stuff is used instead of penicillin.

Answer B: The length of time other antibiotics have been in use is not relevant. This does nothing to help the claim that abandoning penicillin in favor of new, stronger antibiotics will lead to more diseases caused by drug-resistant bacteria.

Answer C: Price and profit margin are a distraction here, mentioned briefly in the stimulus but in no way related to the conclusion that there will be more of certain diseases. Increased profit is one motive for abandoning penicillin, but does nothing to explain the effect of the switch.

Answer D: Again, financial issues (expense here, profit in the previous answer) are not relevant to a discussion of the practical results of making the change. We are only concerned with helping prove that there will be a spike in diseases caused by drug-resistant bacteria. The issue of expense is a distraction, making this answer a loser.

Answer E: This answer could seem attractive if you aren't careful. In fact, this is an opposite answer. If certain bacteria that penicillin did not kill will now be killed by the modern antibiotics, that would weaken the argument that there will be more disease caused by drug-resistant bacteria. If this answer were reversed, saying that some bacteria not resistant to penicillin is resistant to modern antibiotics, then we would be helping our case, as those bacteria might flourish once penicillin is abandoned. Careful reading here should help you avoid this trick answer and identify it as the loser it is.

Get the most out of your LSAT Prep Plus subscription.

Analyze and track your performance with our Testing and Analytics Package.